LSAT and Law School Admissions Forum

Get expert LSAT preparation and law school admissions advice from PowerScore Test Preparation.

 stsai
  • Posts: 27
  • Joined: Nov 05, 2011
|
#2643
"An editor is compiling a textbook..."
This LR looks like a logic game question!
I did not know how to tackle this question other than jotting down these two rules:
L,K,J (not all)
k-->J
After that I was stuck...
 Jon Denning
PowerScore Staff
  • PowerScore Staff
  • Posts: 904
  • Joined: Apr 11, 2011
|
#2645
Thanks for the question. Let's just think of the groups that could be formed:

Singles: L or J (no K alone because it requires J)
Doubles: KJ or LJ (no KL because that would be KJL)

Based on those four possible selections we should be able to quickly find the answer that must be true:

A. True. We cannot have K and L because that would mean K, L, and J (K --> J) and we can't have all three.
B. Not true. Can have double groups.
C. Not true. Can have KJ
D. Not true. Can have L alone
E. Not true. Can have KJ with no L

I hope that helps!
 stsai
  • Posts: 27
  • Joined: Nov 05, 2011
|
#2646
Yes it does. Thanks, Jon!
 SLF
  • Posts: 40
  • Joined: Oct 01, 2013
|
#12925
The stimulus says precisely: "The book will contain essayS by L, K, or J, but it will not contain essayS by all three."

When I read this, I took it to mean that the book would have 2 essays...given the plurality of essayS.

As a result, I created the following conditionals:

K :arrow: J and !L

Which for me then led to the inference

L :arrow: K or J

And the corresponding contrapositives...none of which enabled me to answer the question.

So, how did you know to make the jump from essayS...and translate the stimulus into: "The book may contain a single essay...or it may contain a double group"?

That is, how did you know that essayS did not really mean essayS and that essayS included essay?

It is "misleading" questions like this that consume a lot of time for me...so any help you can offer would be greatly appreciated.
 SLF
  • Posts: 40
  • Joined: Oct 01, 2013
|
#12926
One other thing...

As I was processing this question...and reading through the answer choices, 'A' and 'D' reminded me of the "uniqueness" rule that the Logical Reasoning Bible talks about...these two answer choices struck me as saying the same thing...just worded differently...and so I crossed out both of them.

Any comments on this?
User avatar
 Dave Killoran
PowerScore Staff
  • PowerScore Staff
  • Posts: 5853
  • Joined: Mar 25, 2011
|
#12935
Hi SLF,

There are a couple different issues cropping up here, so let's take a look at all of them. I'll start by explaining this question from the angle in which LSAC sees this problem.

LSAC would make the point that there is a difference between essays and authors. Meaning, it is possible for a single author to have written multiples essays in the book. Thus, when you break down the stimulus, there is an opening there so that only one of the three authors is selected (and not two, as you thought). Before looking at that, let me just say that it is a tricky word game they play, but perhaps not unexpected since this is question #19, and #19s are often tough (not always, but some of the hardest questions out there have been #19s—probably because this is the point in the section where many people begin to feel serious time pressure).

Let's look at the stimulus more closely. The first sentence shows plurality on both essays and authors: "An editor is compiling a textbook containing essays by several different authors." This could mean one or more essays by each author are included. Note also that this simply says "authors," and the reference in the next line mentions three specific authors, but based on this sentence, there could be more authors in play. Thus, when the next sentence says, "The book will contain essays by Lind, Knight, or Jones...," it's possible that just one is selected, but that multiple essays from that person are included. This is how they get past the "essays" objection you raised, and it then cancels your question about the Uniqueness Theory applying to the two answers.

One way around this issue is to realize that under your interpretation of the scenario—that exactly two from the group L, K, or J selected—there are multiple correct answers (A, D, and E are all correct when exactly two are selected). The rules of the section would tell you that is impossible, and thus you would have to go back to the stimulus to re-evaluate your analysis.

Final note, one issue you ran into in your analysis was that this statement is misleading: "L :arrow: K or J." LK is ruled out by the stimulus, because if you choose L and K, you then get J, which violates the stimulus statement that you can't have all three.

Tricky no doubt, and annoying as well.

Please let me know if that helps. Thanks!
 SLF
  • Posts: 40
  • Joined: Oct 01, 2013
|
#12939
Thank you. That is very helpful.

Get the most out of your LSAT Prep Plus subscription.

Analyze and track your performance with our Testing and Analytics Package.